Đến nội dung

Noobmath nội dung

Có 19 mục bởi Noobmath (Tìm giới hạn từ 09-05-2020)


Sắp theo                Sắp xếp  

#410966 Tìm $a>1$ để dãy sau hội tụ $$\left\{\...

Đã gửi bởi Noobmath on 07-04-2013 - 09:45 trong Dãy số - Giới hạn

1) $x_2=\frac{1}{2} =\frac{1}{2!} , x_3=\frac{1}{6}=\frac{1}{3!}$

Ta chứng minh quy nạp : $ x_n = \frac{1}{n!} $ với mọi $n\geq 2$ 

Thật vậy giả sử $x_{n-1}=\frac{1}{(n-1)!} , x_n=\frac{1}{n!} (n \geq 3)$

Khi đó $n(n+1)x_{n+1}=\frac{n(n-1)}{n!}-\frac{n-2}{(n-1)!}=\frac{1}{(n-1)!}$ 

Suy ra $x_{n+1}=\frac{1}{(n+1)!}$ (đpcm)

Vậy $ \frac{x_1}{x_2}+\frac{x_2}{x_3}+...+\frac{x_{50}}{x_{51}}=\frac{1}{2}+3+4+...+51= ... $




#410965 Vui cùng giới hạn 2

Đã gửi bởi Noobmath on 07-04-2013 - 09:39 trong Dãy số - Giới hạn

Xét $f(x)=7-\log_{3}(x^2+11)$

$f'(x)=\frac{2x}{(x^2+11)\ln 3} \Rightarrow |f'(x)|<\frac{1}{2} $ với mọi $x \in \mathbb{R} $

Ta có theo định lý Lagrange : 

$|x_{n+1}-4|=|7-\log_{3}(x^2+11)-4|=|f(x_n)-f(4)|=|f'(c)|.|x_n-4|<\frac{1}{2}|x_n-4|<...<(\frac{1}{2})^n|x_1-4|$

Mà $\lim (\frac{1}{2})^n|x_1-4|=0 \Rightarrow \lim (x_n-4)=0 $ hay $\lim x_n = 4 $ 




#410845 Cho $n \in N^*$. Chứng minh rằng:$$\lim_{x...

Đã gửi bởi Noobmath on 06-04-2013 - 20:20 trong Dãy số - Giới hạn

$\frac{x^n-\sin^n x }{x^{n+2}} = \frac{(x-\sin x)(x^{n-1}+x^{n-2}\sin x +...+x\sin^{n-2}x+\sin^{n-1}x)}{x^{n+2}}=\frac{x-\sin x}{x^3}.(1+\frac{\sin x}{x}+\frac{\sin^2 x }{x^2}+...+\frac{\sin^{n-1}x}{x^{n-1}})$

Đến đây ta có 2 giới hạn sau : 

1) $\lim_{x \to 0} \frac{x- \sin x }{x^3} = \frac{1}{6}$

2) $\lim_{x \to 0}\frac{\sin x}{x} = 1 $

Với các giới hạn trên ta có điều phải chứng minh . 




#408118 $x_{n}=2\sum_{k=1}^{n}\frac...

Đã gửi bởi Noobmath on 26-03-2013 - 19:51 trong Dãy số - Giới hạn

Mình chỉ chứng minh được $ \lim x_n = \ln 2 $ ( mà cũng không chắc là có đúng không)

Ta có : $\frac{1}{n+k-1}=\frac{2}{2n+2k-2}>\frac{2}{2n+2k-1}>\frac{2}{2n+2k}=\frac{1}{n+k}$ 

$\Rightarrow \sum_{k=1}^{n} \frac{1}{n+k-1}>\sum_{k=1}^{n} \frac{2}{2n+2k-1} > \sum_{k=1}^{n} \frac{1}{n+k}$

Mà $ \lim \sum_{k=1}^{n} \frac{1}{n+k-1}=\lim \frac{1}{n} \sum_{k=1}^{n} \frac{1}{1+\frac{k-1}{n}} = \int_{0}^{1} \frac{dx}{x+1}=\ln 2$ 

Tương tự $\lim \sum_{k=1}^{n} \frac{1}{n+k} = \int_{0}^{1} \frac{dx}{x+1}=\ln 2$ 

Vậy $ \lim x_n = \lim \sum_{k=1}^{n} \frac{2}{2n+2k-1} = \ln 2$ (Theo nguyên lý kẹp) 




#406866 $u_{n+1}=2u_n+cos(u_n)$

Đã gửi bởi Noobmath on 21-03-2013 - 21:56 trong Dãy số - Giới hạn

Làm như tnay k biết đúng ko , nếu sai thì mọi người thông cảm . ^_^

Xét $f(x)=x+ \cos x$ khả vi trên $\mathbb{R}$

$f'(x)=1- \sin x \geq 0 , \forall x \in \mathbb{R}$

Vậy $f(x)$ đồng biến trên $\mathbb{R}$ suy ra $f(x)=0$ có nhiều nhất 1 nghiệm.

Mặt khác : $f(0)=1 , f(-\frac{\pi}{2})=-\frac{\pi}{2}$ ; $f(x)$ liên tục .

Suy ra $f(x)$ có duy nhất 1 nghiệm thuộc $(-\frac{\pi}{2};0)$

Gọi nghiệm là L.

+) $a=L \Rightarrow u_n = L , \forall n $ hay $ \lim u_n = L$ 

+) $a>L \Rightarrow u_2 > u_1 ... u_n $ tăng.

Giả sử $u_n$ bị chặn trên suy ra có giới hạn hữu hạn $\lim u_n = u$

Suy ra $u=L$ ( vô lý) . Vậy $ \lim u_n = +\infty $ 

+) $a<L$ tương tự $\lim u_n = -\infty $ 




#406327 $f(x)=\left\{\begin{matrix} x^2sin\fr...

Đã gửi bởi Noobmath on 19-03-2013 - 20:34 trong Hàm số - Đạo hàm

Muốn chứng minh $\lim_{x \to 0} \cos \frac{1}{x}$  ko tồn tại cũng đơn giản thôi , dùng định nghĩa giới hạn của hàm số.

Định nghĩa : $ \lim_{x \to x_0} f(x) = L \Leftrightarrow \forall (x_n) : \lim x_n = x_0$ thì $\lim f(x_n) = L$ 

Vậy ta sẽ chọn các dãy $x_n$ phù hợp để chứng minh. 

Dãy đầu tiên ta chọn $x_n = \frac{1}{2n\pi}$ ( hiển nhiên $\lim x_n =0$) 

Khi đó $\cos \frac{1}{x_n} =\cos (2n\pi)=1$ hay $\lim \cos \frac{1}{x_n}=1$ 

Tương tự chọn $x_n = \frac{1}{(2n+1)\pi}$ thì $\lim \cos \frac{1}{x_n}=-1$ 

Vậy theo định nghĩa thì suy ra không tồn tại $\lim_{x \to 0} \cos \frac{1}{x}$




#403990 tìm $\lim_{x \to 0}\frac{\sqrt{1...

Đã gửi bởi Noobmath on 11-03-2013 - 13:01 trong Dãy số - Giới hạn

Chỉ cần dùng 1 biến đổi đơn giản thui mà : cộng trừ tử số với $(x+1)$
Ta có : $\sqrt{2x+1} - (x+1) = \frac{-x^2}{\sqrt{2x+1}+(x+1)}$
$(x+1)- \sqrt[3]{3x+1} = \frac{x^3+3x^2}{(x+1)^2+(x+1)\sqrt[3]{3x+1}+\sqrt[3]{3x+1}^2}$
Chia $x^2$ xuống thì sẽ mất dạng vô định rùi cho $x=0$ vô là xong



#402717 $f(x)=a.f(x)=a.\sin5x+b.+b.\cos5x+c.+c.\sin x+d.+d.\...

Đã gửi bởi Noobmath on 07-03-2013 - 13:38 trong Phương trình, Hệ phương trình Lượng giác

$f(x)$ liên tục trên $\mathbb{R}$
Mà $f(0)=b+d , f(\pi)=-b-d \Rightarrow f(0)f(\pi)=-(b+d)^2 \leq 0 $
Vậy $\exists a \in [0;\pi] : f(a)=0$



#400665 Tìm giới hạn dãy số cho bởi $u_{n-1}=n^2(u_{n-1}-u_...

Đã gửi bởi Noobmath on 28-02-2013 - 13:47 trong Dãy số - Giới hạn

Có thể quy nạp thẳng lun : $ u_n = \frac{2011(n+1)}{2n} $
Hoặc nếu thích rỏ ràng thì , từ công thức truy hồi suy ra : $n^2u_n=(n^2-1)u_{n-1}$
Hay $n^2u_n=(n-1)u_{n-1}.(n+1)$
Đặt $nu_n = v_n$
Suy ra ( dễ dàng chứng minh $u_n$ khác 0 dẫn đến $v_n$ khác 0)
$\frac{v_{n}}{v_{n-1}} = \frac{n+1}{n}$
Giảm chỉ số xuống , nhân vế với vế của các đẳng thức thì suy ra :
$\frac{v_n}{v_1} = \frac{n+1}{2}$
( $v_1 = 2011$ ) suy ra : $u_n = \frac{2011(n+1)}{2n}$
Đến đây dễ dàng suy ra $ \lim u_n =\frac{2011}{2}$



#393048 tìm a để $u_n$ hội tụ

Đã gửi bởi Noobmath on 04-02-2013 - 10:19 trong Dãy số - Giới hạn

Cho dãy $u_n$ xác định bởi : $u_1 =a , u_{n+1} = u_n^2+\frac{u_n}{2}$
Tìm tất cả các giá trị của a để cho dãy $u_n$ hội tụ



#386013 VMO 2013 - Bài 2. Dãy số Giới hạn

Đã gửi bởi Noobmath on 12-01-2013 - 19:37 trong Dãy số - Giới hạn

Bài này đâu cần cm chặt đến thế, chỉ cần cm $0< a_n< 3$ (dùng đạo hàm cm $0< a_n$ trước sau đó có $a_n<3$)

Nếu $a_n<3$ thì dãy mới hội tụ còn cái bất đẳng thức kia phải chặt thì mới tìm được giới hạn



#382790 Tính $a_{2013}$

Đã gửi bởi Noobmath on 01-01-2013 - 23:23 trong Dãy số - Giới hạn

1. Cho dãy : $a_{n+3}=2a_{n+2}-2a_{n+1}+a_n$
Biết $a_{15}=82 , a_{20}=1 , a_{30}=100$ . Tính $a_{2013}$
2. Tính $ \lim_{n \to \infty} \int_{0}^{\frac{1}{n}} x^{x+1}dx $



#381419 $f(f(x)+y)=xf(1+xy)$

Đã gửi bởi Noobmath on 28-12-2012 - 23:13 trong Phương trình hàm

Thay $x=0,y=y-f(0)$ ta được f(y)=0. Dễ thấy hàm này thỏa

$x,y>0$ ???



#381410 $f(f(x)+y)=xf(1+xy)$

Đã gửi bởi Noobmath on 28-12-2012 - 23:04 trong Phương trình hàm

Tìm tất cả các hàm số $f:\mathbb{R}^{*} \to \mathbb{R}^{*}$

$f(f(x)+y)=xf(1+xy) , \forall x,y \in \mathbb{R}^{*}$




#381407 $P=\frac{1+a+b+c}{3+2a+b}-\frac{c...

Đã gửi bởi Noobmath on 28-12-2012 - 22:59 trong Bất đẳng thức và cực trị

Giả sử phương trình $x^3-ax^2+bx-c=0$ có 3 nghiệm thực dương ( không nhất thiết phải khác nhau ) . Hãy tìm GTNN của biểu thức:

$P=\frac{1+a+b+c}{3+2a+b}-\frac{c}{b}$




#381399 Tìm Max P = $a^{-1}+b^{-1}+c^{-1}$

Đã gửi bởi Noobmath on 28-12-2012 - 22:36 trong Bất đẳng thức và cực trị

Đề yêu cầu tìm $\max$ mà bạn

Bài này sao có max đc , cho a dần đến 0 thì P tiến đến vô cực



#381391 $\lim_{m \to \infty} m \sum_{k=m+1...

Đã gửi bởi Noobmath on 28-12-2012 - 22:14 trong Dãy số - Giới hạn

Dễ dàng chứng minh công thức tổng quát của $a_n$ bằng quy nạp : $a_n=\frac{1}{n^2} , \forall n \geq 1 $
Vậy : $\lim_{m \to \infty} m \sum_{k=m+1}^{2m}a_k=\lim_{m \to \infty} m \sum_{k=m+1}^{2m}\frac{1}{k^2}=\lim_{m \to \infty} \frac{1}{m} \sum_{k=1}^{m} \frac{1}{(\frac{k}{m}+1)^2}=\int_{0}^{1} \frac{1}{(x+1)^2} dx = \frac{1}{2}$



#365602 $f(n) \equiv n^2$

Đã gửi bởi Noobmath on 28-10-2012 - 19:28 trong Phương trình hàm

$f(1)=1 , f(2)=4 , f(2^n) = 4^n $
Xét $m \in \mathbb{N}^* , m \geq 3 $
$\forall t \in \mathbb{N}^* , \exists k : 2^k \leq m^t <2^{k+1}$
$\Rightarrow \left\{\begin{matrix} 4^k=f(2^k) \leq f(m)^t=f(m^t) < f(2^{k+1})=4^{k+1} \\ 4^k \leq (m^2)^t < 4^{k+1}\end{matrix}\right.$
$\Rightarrow \frac{1}{4}<\left (\frac{f(m)}{m^2}\right )^t<4$
Từ đây suy ra $\frac{f(m)}{m^2}=1$ .
( vì nếu khác 1 thì lim của nó sẽ tiến đến 0 hoặc dương vô cùng và không thỏa mãn bất đẳng thức trên với mọi $t$)
Hay $f(m) \equiv m^2 $
ĐPCM



#365597 Tìm n sao cho $a_n \in Z $

Đã gửi bởi Noobmath on 28-10-2012 - 19:13 trong Dãy số - Giới hạn

Đặt $b_n = na_n \Rightarrow \left\{\begin{matrix} b_1=b_2 = 2 \\ b_{n+2}=nb_{n+1}+nb_n-2n+1\end{matrix}\right.$
Đặt $c_n=b_n -1 \Rightarrow \left\{\begin{matrix} c_1=c_2 = 1 \\ c_{n+2}=nc_{n+1}+nc_n(*)\end{matrix}\right.$
Từ (*) suy ra : $c_{n+2}-(n+1)c_{n+1}=-(c_{n+1}-nc_n)= ... = (-1)^n(c_2-1.c_1)=0$
$\Rightarrow c_{n+2}=(n+1)c_{n+1}=...=(n+1)!$
$\Rightarrow a_n=\frac{(n-1)!+1}{n}$
Suy ra theo định lý Wilson : $a_n \in \mathbb{Z} \Leftrightarrow n \in \mathbb{P}$